Difference between revisions of "2022 AMC 10A Problems/Problem 7"

(See Also)
(Solution)
Line 5: Line 5:
 
<math>\textbf{(A) } 3 \qquad \textbf{(B) } 6 \qquad \textbf{(C) } 8 \qquad \textbf{(D) } 9 \qquad \textbf{(E) } 12</math>
 
<math>\textbf{(A) } 3 \qquad \textbf{(B) } 6 \qquad \textbf{(C) } 8 \qquad \textbf{(D) } 9 \qquad \textbf{(E) } 12</math>
  
== Solution ==  
+
== Solution 1 ==  
 
Note that
 
Note that
 
<cmath>\begin{align*}
 
<cmath>\begin{align*}
Line 20: Line 20:
  
 
~MRENTHUSIASM
 
~MRENTHUSIASM
 +
 +
== Solution 2 ==
 +
==Problem==
 +
 +
The least common multiple of a positive integer <math>n</math> and <math>18</math> is <math>180</math>, and the greatest common divisor of <math>n</math> and <math>45</math> is <math>15</math>. What is the sum of the digits of <math>n</math> ?
 +
 +
 +
<math>\textbf{(A)} 3  \textbf{ (B)} 6  \textbf{ (C)} 8  \textbf{ (D)} 9  \textbf{ (E)} 12</math>
 +
 +
== Solution 2 ==
 +
Since the <math>lcm</math> contains only factors of <math>2</math>, <math>3</math>, and <math>5</math>, <math>n</math> cannot be divisible by any other prime.
 +
Let n = <math>2^a</math> <math>3^b</math> <math>5^c</math>, where <math> a</math> ,<math>b</math>, and <math>c</math> are nonnegative integers.
 +
We know that <math>lcm(n, 18)</math> = <math>lcm(n, 3^2 * 2)</math> = <math>lcm(2^a * 3^b * 5^c, 3^2 * 2)</math> =
 +
<math>lcm(2^ {max(a,1)} \cdot 3^ {max(a,2)} \cdot 5^b)</math> = <math>180</math> = <math>2^2 \cdot 3^2 \cdot 5.</math>
 +
Thus
 +
 +
<math> (1) max(a,1) = 2 -> a = 2 </math>
 +
 +
<math> (2) max(a,2) = 2 -> 0 <= a <= 2</math>
 +
 +
<math>(3) c = 1.</math>
 +
From the gcf information, <math>gcf(n,45)</math> = gcf(n, <math>3^2 \cdot 5</math>) = <math>gcf(2^a \cdot 3^b \cdot 5^c, 3^2 \cdot 5) </math>
 +
= <math>3^min(b,2) \cdot 5^min(c,1) = 15</math>
 +
This means, that since c = 1, 3^min(b,2) \cmath 5 = 15, so min(b,2) = 1 and b = 1.
 +
Hence, multiplying using a = 2, b = 1, c = 1 gives n = 60 and the sum of digits is hence 6.
  
 
== See Also ==
 
== See Also ==

Revision as of 01:38, 12 November 2022

Problem

The least common multiple of a positive divisor $n$ and $18$ is $180$, and the greatest common divisor of $n$ and $45$ is $15$. What is the sum of the digits of $n$?

$\textbf{(A) } 3 \qquad \textbf{(B) } 6 \qquad \textbf{(C) } 8 \qquad \textbf{(D) } 9 \qquad \textbf{(E) } 12$

Solution 1

Note that \begin{align*} 18 &= 2\cdot3^2, \\ 180 &= 2^2\cdot3^2\cdot5, \\ 45 &= 3^2\cdot5 \\ 15 &= 3\cdot5. \end{align*} From the least common multiple condition, we conclude that $n=2^2\cdot 3^k\cdot5,$ where $k\in\{0,1,2\}.$

From the greatest common divisor condition, we conclude that that $k=1.$

Therefore, we have $2^2\cdot 3^1\cdot5=60.$ The sum of its digits is $6+0=\boxed{\textbf{(B) } 6}.$

~MRENTHUSIASM

Solution 2

Problem

The least common multiple of a positive integer $n$ and $18$ is $180$, and the greatest common divisor of $n$ and $45$ is $15$. What is the sum of the digits of $n$ ?


$\textbf{(A)} 3  \textbf{ (B)} 6  \textbf{ (C)} 8  \textbf{ (D)} 9  \textbf{ (E)} 12$

Solution 2

Since the $lcm$ contains only factors of $2$, $3$, and $5$, $n$ cannot be divisible by any other prime. Let n = $2^a$ $3^b$ $5^c$, where $a$ ,$b$, and $c$ are nonnegative integers. We know that $lcm(n, 18)$ = $lcm(n, 3^2 * 2)$ = $lcm(2^a * 3^b * 5^c, 3^2 * 2)$ = $lcm(2^ {max(a,1)} \cdot 3^ {max(a,2)} \cdot 5^b)$ = $180$ = $2^2 \cdot 3^2 \cdot 5.$ Thus

$(1) max(a,1) = 2 -> a = 2$

$(2) max(a,2) = 2 -> 0 <= a <= 2$

$(3) c = 1.$ From the gcf information, $gcf(n,45)$ = gcf(n, $3^2 \cdot 5$) = $gcf(2^a \cdot 3^b \cdot 5^c, 3^2 \cdot 5)$ = $3^min(b,2) \cdot 5^min(c,1) = 15$ This means, that since c = 1, 3^min(b,2) \cmath 5 = 15, so min(b,2) = 1 and b = 1. Hence, multiplying using a = 2, b = 1, c = 1 gives n = 60 and the sum of digits is hence 6.

See Also

2022 AMC 10A (ProblemsAnswer KeyResources)
Preceded by
Problem 6
Followed by
Problem 8
1 2 3 4 5 6 7 8 9 10 11 12 13 14 15 16 17 18 19 20 21 22 23 24 25
All AMC 10 Problems and Solutions
2022 AMC 12A (ProblemsAnswer KeyResources)
Preceded by
Problem 3
Followed by
Problem 5
1 2 3 4 5 6 7 8 9 10 11 12 13 14 15 16 17 18 19 20 21 22 23 24 25
All AMC 12 Problems and Solutions

The problems on this page are copyrighted by the Mathematical Association of America's American Mathematics Competitions. AMC logo.png